LSAT and Law School Admissions Forum

Get expert LSAT preparation and law school admissions advice from PowerScore Test Preparation.

 curiosity
  • Posts: 17
  • Joined: Jul 14, 2014
|
#15328
I now understand why B is the correct answer through process of elimination, but I don't understand how the fewer children being born to women over 40 matters, particularly because there could still be objectively more ambidexterous children born to these women. Why does that affect the conclusion at all? Please help me understand this!

Thanks!
 Lucas Moreau
PowerScore Staff
  • PowerScore Staff
  • Posts: 216
  • Joined: Dec 13, 2012
|
#15335
Hello, curiosity,

The fact of there being fewer children born to women over 40 matters a lot! Since the conclusion is that women over 40 are birthing objectively (not proportionately) more ambidextrous children than younger women, if the total numbers looked like this:

Children born to women over 40 last year: 5,000
Children born to women under 40 last year: 10,000,000

Then that conclusion starts looking a little unlikely! 8-) The author bases this conclusion solely on the "older birth :arrow: more likely to be difficult" and "more difficult birth :arrow: more ambidextrous children" premises, without considering the respective numbers of children born to each group of women.

Hope that helps,
Lucas Moreau
 curiosity
  • Posts: 17
  • Joined: Jul 14, 2014
|
#15358
Thanks, that does clear it up. I think I was considering it as a proportion instead of an objective number.
 PB410
  • Posts: 39
  • Joined: Apr 01, 2017
|
#36171
Hi,
I came across answer B, yet I was a little hesitant because I thought the the answer choice might be assuming the fact that "fewer children are born to women over 40 that to women under 40". The stimulus does not state anything that could lead to this thought and I was skeptical the test makers would be feeding to our assumptions. Is there any explanation you can offer on a broader scope of when it is appropriate to rely on outside details?
 Francis O'Rourke
PowerScore Staff
  • PowerScore Staff
  • Posts: 471
  • Joined: Mar 10, 2017
|
#36206
Hi PB,

The amount of children born to women over vs under 40 is not out of scope for this stimulus. The author discusses the rates of 'difficult births' for women under vs over 40, so the absolute number of births for women in these two age groups is extremely pertinent to the argument. Whenever an author discusses rates, the absolute number of occurrences may be relevant to a discussion why a conclusion is flawed.
User avatar
 simonsap
  • Posts: 34
  • Joined: Jun 14, 2021
|
#87912
CAUSE A (Injury) --> Difficult Birth --> Ambidextrous
CAUSE B (Disease) --> Difficult Birth --> Ambidextrous
CAUSE C (Small hips) --> Difficult Birth --> Ambidextrous
CAUSE D (AGE 40+) --> Difficult Birth --> Ambidextrous

conclusion: most Ambidextrous kids are born to Old mothers.

Huh? How did it reach that conclusion? There isn't sufficient evidence to prove that conclusion. No numbers/distribution of causes was provided. The argument biases majority of the weight on one cause (Age) without telling us what proportion of the total cases are caused by Age.
 Rachael Wilkenfeld
PowerScore Staff
  • PowerScore Staff
  • Posts: 1358
  • Joined: Dec 15, 2011
|
#88264
Hi Simonsap

Our stimulus tells us that

1. Mother's over 40 are more likely to have difficult births
2. Children from difficult births are more likely to be ambidextrous
3. Other causes of ambidexterity are independent of age of the mother
3. Therefore, there are more ambidextrous people born to mothers over 40 than under 40.

All the other causes you list are independent of age, so for the purposes of the argument they give us, we can ignore them. If there are 5 small hips difficult births to women under 40, we expect 5 for women over 40. That's what the clause about the other causes being independent from age indicates. We can't argue with the information given in the stimulus.

We can then treat all the other causes as irrelevant, and focus only on the issue of mother's age.

Hope that helps!
User avatar
 teddykim100
  • Posts: 43
  • Joined: Jan 10, 2022
|
#98239
Hello,

this is more of an approach question but

the way I think about flaw questions is that they either have one of two problems
1. the premise and conclusion don't match up (term shift)
2. or the author did not consider other factors

I did get B as the correct answer, but only because I thought the author neglected to consider the idea that more births could occur with women under 40 than over.

would this be an appropriate alternate approach? Or does the term shift take priority here?
User avatar
 teddykim100
  • Posts: 43
  • Joined: Jan 10, 2022
|
#98240
to follow up with the above:

for instance, other causes of ambidexterity might not be related to the mother's age (the author rules that out), but perhaps a factor like genetics is. Perhaps if the mother is a smoker is, perhaps cancer, etc. . .
 Rachael Wilkenfeld
PowerScore Staff
  • PowerScore Staff
  • Posts: 1358
  • Joined: Dec 15, 2011
|
#98244
Hi Teddy

Your overall approach sounds good here! It's true that the issue here is that we don't know about population sizes in terms of the number of babies born to mothers over or under 40. That's the flaw you recognized, and that answer choice (B) describes.

One caution for your second post: genetics is another cause, not different from a cause. It's an explanation in the same way age is an explanation. Don't treat genetics as different than other causes unless the stimulus specifies it is different in a critical way. Otherwise, it's logically the same as other causes.

Great work!

Get the most out of your LSAT Prep Plus subscription.

Analyze and track your performance with our Testing and Analytics Package.